ChaseDream
搜索
返回列表 发新帖
00:00:00

The higher the level of certain vitamins and minerals in the bloodstream, the better a person's lung function, as measured by the amount of air the person can expel in one second. The lung function of smokers is significantly worse, on average, than that of nonsmokers. Clearly, therefore, one way for smokers to improve their lung function is for them to increase their intake of foods that are rich in these helpful vitamins and minerals.

Which of the following is an assumption on which this argument depends?

正确答案: E

更多相关帖子

524

帖子

15

好友

4712

积分

ChaseDream

注册时间
2003-03-17
精华
8
解析
查看: 3188|回复: 8
打印 上一主题 下一主题

OG的一道assumption逻辑题,一直搞不清楚

[复制链接]
跳转到指定楼层
楼主
发表于 2012-3-21 16:53:47 | 只看该作者 回帖奖励 |倒序浏览 |阅读模式
The higher the level of certain vitamins and minerals in the bloodstream, the better a person's lung function, as measured by the amount of air the person can expel in one second.The lung function of smokers is significantly worse, on average, than that of nonsmokers. Clearly, therefore, one way for smokers to improve their lung function is for them to increase their intake of foods that are rich in these helpful vitamins and minerals.

Which of the following is an assumption on which this argument depends?



(A) Smokers are less likely than nonsmokers to have diets that are rich in vitamins and minerals.

(B) The lung function of smokers whose diets are rich in those vitamins and minerals is generally better than that of nonsmokers with comparable diets.

(C) People whose diets are deficient in those vitamins and minerals do not typically have other health problems[c4] in addition to diminished lung function.

(D) Stopping smoking will not typically improve lung function more than any diet changes can.

(E) Smoking does not introduce into the body chemicals that prevent the helpful vitamins and minerals from entering the bloodstream.

其实不懂为什么是选A,做的时候感觉很难选,感觉没一个对的。希望nn帮忙解释下
收藏收藏 收藏收藏
沙发
发表于 2012-3-21 17:06:51 | 只看该作者
同问。。。没有一个可以选
板凳
发表于 2012-3-21 18:13:39 | 只看该作者
说一下我的看法。。也不知道对不对
conclusion:one way for smokers to improve their lung function is for them to increase their intake of foods that are rich in these helpful vitamins and minerals.
premise:The higher the level of certain vitamins and minerals in the bloodstream, the better a person's lung function
要从premise推出conclusion就是直接认为higher the level of certain vitamins and minerals in the bloodstream会导致 the better a person's lung function
实质上这两个东西只是两个相互有关联的现象,不一定是有因果关系,就算有因果关系,也不一定是A→B

(A) Smokers are less likely than nonsmokers to have diets that are rich in vitamins and minerals.(因为文中有说The lung function of smokers is significantly worse, on average, than that of nonsmokers  A选项应该是加强了rich in vitamins and minerals→better lung function的可能性)
(B) The lung function of smokers whose diets are rich in those vitamins and minerals is generally better than that of nonsmokers with comparable diets.(这个我觉得直接out of scope了,因为这个是在比diets一样的情况下的smoker和nonsmoker的lung function)
(C) People whose diets are deficient in those vitamins and minerals do not typically have other health problems[c4] in addition to diminished lung function.(这个也out of scope这个直接将other health problems了)
(D) Stopping smoking will not typically improve lung function more than any diet changes can.(这个也是out of scope,它讲的是stop smoking)
(E) Smoking does not introduce into the body chemicals that prevent the helpful vitamins and minerals from entering the bloodstream.(这个选项我觉得很attractive,它讲是smoking不会产生阻止v和m进入bloodstream的物质。但是实质上v和m就算进入了血液,它也不一定会使lung function变强,所以这又回到了它们之间的因果关系上去了)
地板
发表于 2012-3-21 19:36:28 | 只看该作者
亲。。。。。。。真服了你了。。。折腾死我了。。。这道题选E啊。。。。害的我看了40多分钟。。。。
还有就是你发帖之前麻烦一搞清楚真正的答案是啥,二就是先在论坛搜索一下有没有人问你的问题,一般都会有人问过的。。这就有一个。。。
http://forum.chasedream.com/GMAT_CR/thread-681210-1-1.html?postid=16048321

那我说下我的看法。

首先要明白题目问什么,明显题目问的是结论正确的必要条件是什么(注意不是充分)就是说C(结论)-> (必要条件),或者你运用逆否命题,就是非P->非C

所以做题方法就是各否定五个选项,能推出题干中原结论错误的就是正确选项

A :否定A就是说吸烟者可能比非吸烟者吃的v和m多或者一样,但这并不能否定结论,比如由于你吸烟导致v和m的吸收更差,那你就要吃更多的v和m
B:否定得出同等情况下吸烟者肺功能不比非吸烟者好,这更能推出结论了。。。所以错
C:否定后得出:有其他健康问题。但是那又怎样,不妨碍你推出结论。
D:否定后你得出一种可以提高肺功能的方法,但是原题的结论是多吃V和M是一种有效地方法,没说是唯一的,所以不妨碍结论正确性。
E:若否定E,则得出吸烟会阻止身体吸收V和M,所以多吃含V和M多的食品不能保障你就能提高肺功能,所以否定E得不出原结论。。。。也就是说为了使原结论正确E必须正确,所以选E
5#
发表于 2012-3-21 19:38:11 | 只看该作者
哦,再说下,楼上貌似搞错了问题体型(我也没仔细看)

推荐看下这个帖子,用处很大的哦

http://forum.chasedream.com/gmat_cr/thread-569879-1-1.html
6#
发表于 2012-3-21 19:54:32 | 只看该作者
TAT 原来如此
其实我在写A的时候也觉得由A其实也得不出A→B的可能性
E的话我搞反先后了,应该是
(E) Smoking does not introduce into the body chemicals that prevent the helpful vitamins and minerals from entering the bloodstream.
(它讲是smoking不会产生阻止v和m进入bloodstream的物质。就算A能推B但是v和m进不了血液,lung function还是不会提高。不过A→B也应该算是这个argument的一个assumption吧,只不过没列在选项里)
7#
 楼主| 发表于 2012-3-28 14:20:38 | 只看该作者
这是我第二次做这个题,是做别人弄好的prep07,没想到答案是错的,不好意思。那时想拿起OG再看看,却找不到,就直接贴上去问了。
老实说:我不知道怎么在论坛里搜索已经问过的题目~~
8#
发表于 2012-3-29 17:38:31 | 只看该作者
晕。。。网站右上角偏下一点的位置有个“搜索论坛”,一般搜已经问过的问题就选“主题内容”(要点进去,就是这个http://forum.chasedream.com/search.html?fid=0&mode=1 )就行了。。。
9#
发表于 2012-5-10 14:19:00 | 只看该作者
您好想问一下是不是假设题就是找结论的必要条件啊??
亲。。。。。。。真服了你了。。。折腾死我了。。。这道题选E啊。。。。害的我看了40多分钟。。。。
还有就是你发帖之前麻烦一搞清楚真正的答案是啥,二就是先在论坛搜索一下有没有人问你的问题,一般都会有人问过的。。这就有一个。。。
http://forum.chasedream.com/GMAT_CR/thread-681210-1-1.html?postid=16048321

那我说下我的看法。

首先要明白题目问什么,明显题目问的是结论正确的必要条件是什么(注意不是充分)就是说C(结论)-> (必要条件),或者你运用逆否命题,就是非P->非C

所以做题方法就是各否定五个选项,能推出题干中原结论错误的就是正确选项

A :否定A就是说吸烟者可能比非吸烟者吃的v和m多或者一样,但这并不能否定结论,比如由于你吸烟导致v和m的吸收更差,那你就要吃更多的v和m
B:否定得出同等情况下吸烟者肺功能不比非吸烟者好,这更能推出结论了。。。所以错
C:否定后得出:有其他健康问题。但是那又怎样,不妨碍你推出结论。
D:否定后你得出一种可以提高肺功能的方法,但是原题的结论是多吃V和M是一种有效地方法,没说是唯一的,所以不妨碍结论正确性。
E:若否定E,则得出吸烟会阻止身体吸收V和M,所以多吃含V和M多的食品不能保障你就能提高肺功能,所以否定E得不出原结论。。。。也就是说为了使原结论正确E必须正确,所以选E
-- by 会员 曳烛 (2012/3/21 19:36:28)

您需要登录后才可以回帖 登录 | 立即注册

Mark一下! 看一下! 顶楼主! 感谢分享! 快速回复:

手机版|ChaseDream|GMT+8, 2024-6-30 18:49
京公网安备11010202008513号 京ICP证101109号 京ICP备12012021号

ChaseDream 论坛

© 2003-2023 ChaseDream.com. All Rights Reserved.

返回顶部